selling price of article

This topic has expert replies
Senior | Next Rank: 100 Posts
Posts: 42
Joined: Sat Apr 12, 2008 8:51 am
Thanked: 1 times

selling price of article

by gibran » Thu May 15, 2008 9:48 am
The selling price of an article is equal to the cost of the article plus the markup. The markup on a certain television set is what percent of the selling price?
(1) The markup on the television set is 25 percent of the cost.
(2) The selling price of the television set is $250.
A. Statement (1) ALONE is sufficient, but statement (2) alone is not sufficient.
B. Statement (2) ALONE is sufficient, but statement (1) alone is not sufficient.
C. BOTH statements TOGETHER are sufficient, but NEITHER statement ALONE is sufficient.
D. EACH statement ALONE is sufficient.
E. Statements (1) and (2) TOGETHER are NOT sufficient.

IMO, it is C. But OA says it is A. Please help to explain with proper steps.
Thanks.

User avatar
Legendary Member
Posts: 543
Joined: Fri Jan 18, 2008 1:01 am
Thanked: 43 times
GMAT Score:580

by codesnooker » Thu May 15, 2008 10:07 am
According to me also, answer should be (C).
What is the source of the question?

Senior | Next Rank: 100 Posts
Posts: 39
Joined: Tue May 06, 2008 11:49 am
Thanked: 1 times

by maria » Thu May 15, 2008 10:26 am
Selling price = M + Cost
Mk = 25% of the cost.
we do not know the cost.

the answer can be E

Senior | Next Rank: 100 Posts
Posts: 39
Joined: Tue May 06, 2008 11:49 am
Thanked: 1 times

by maria » Thu May 15, 2008 11:31 am
any one has an appropriate explanation?

User avatar
Legendary Member
Posts: 543
Joined: Fri Jan 18, 2008 1:01 am
Thanked: 43 times
GMAT Score:580

by codesnooker » Thu May 15, 2008 11:51 am
Dear Maria,

Let me try to explain the method that I have followed.

According to you,

Selling price = M + Cost (CORRECT)

By statement 1,

M = 25% of the cost.

SP = (Cost/4) + Cost

INSUFFICIENT, as it is a linear equation of two unknown variables.

According to 2nd statement:
SP = $250
and SP = M + Cost (as per our forumula)
Therefore, M + Cost = 250

Again INSUFFICIENT as it is a linear equation of two unknown variables.

Now lets take both statements together.

By statement 1,

SP = (Cost/4) + Cost

and By Statement 2, SP = 250

therefore, 250 = (Cost/4) + Cost

Now its is a linear equation of single unknown variable, hence can be solvable. Therefore SUFFICIENT.

Therefore, correct answer should be (C).

Senior | Next Rank: 100 Posts
Posts: 39
Joined: Tue May 06, 2008 11:49 am
Thanked: 1 times

by maria » Thu May 15, 2008 12:18 pm
Codesnooker, I got it. Thank you. :D

Master | Next Rank: 500 Posts
Posts: 343
Joined: Mon Jan 21, 2008 3:28 pm
Thanked: 4 times

by arorag » Thu May 15, 2008 5:38 pm
Guys OA is correct it has to be A.
Let x is cost of tv, markup will be 0.25 x
S.P.= 1.25 x
required info= .25 x/1.25x(100)= 20 %

Senior | Next Rank: 100 Posts
Posts: 39
Joined: Tue May 06, 2008 11:49 am
Thanked: 1 times

by maria » Thu May 15, 2008 8:06 pm
Hmmmm. it appears to have different answers. Any expert helps us to confirm the correct answer to this question?

Junior | Next Rank: 30 Posts
Posts: 14
Joined: Tue Jan 08, 2008 5:48 pm
Thanked: 1 times

by devp » Fri May 16, 2008 7:00 am
The answer A is correct. The problem with Codesnooker's explanation is that M was replaced by C rather than C replaced by M.

Hope that helps.

Senior | Next Rank: 100 Posts
Posts: 58
Joined: Fri Apr 04, 2008 12:36 pm
Thanked: 4 times
GMAT Score:680

by mim3 » Fri May 16, 2008 9:24 am
devp wrote:The answer A is correct. The problem with Codesnooker's explanation is that M was replaced by C rather than C replaced by M.

Hope that helps.
I'm definitely NOT an expert, but it's A. We're looking for a percentage/ratio/proportion between the selling price (P) and mark-up (M). In the stem we're given Price= Cost + Mark-Up (P= C + M)

1. M= .25C
Plugging it into the what we were given:
P= C+.25C
P=1.25C and we established that M=.25C. We can stop there because we have a proportion between P & M: P:M= .25:1.25 = 20%

2. Nothing there.

User avatar
Newbie | Next Rank: 10 Posts
Posts: 1
Joined: Tue Nov 12, 2013 12:52 pm

by breaking bad » Tue Nov 12, 2013 12:56 pm
if we put in numbers it becomes easy !! and A it is.

cost : 100 markup : 25% : 25 sp: 125

now as per the question : 25= x/100*125 =20%

GMAT/MBA Expert

User avatar
GMAT Instructor
Posts: 7222
Joined: Sat Apr 25, 2015 10:56 am
Location: Los Angeles, CA
Thanked: 43 times
Followed by:29 members

by Scott@TargetTestPrep » Fri Aug 03, 2018 2:13 pm
gibran wrote:The selling price of an article is equal to the cost of the article plus the markup. The markup on a certain television set is what percent of the selling price?
(1) The markup on the television set is 25 percent of the cost.
(2) The selling price of the television set is $250.
We can let c = cost of the article and m = the markup and create the expression:

m/(c + m) x 100 = ?

Statement One Alone:

The markup on the television set is 25 percent of the cost.

From statement one, we have m = 0.25c, and thus our expression now is:

m/(c + m) x 100 = ?

0.25c/(c + 0.25c) x 100 = ?

0.25c/(1.25c) x 100 = ?

0.25/1.25 x 100 = 20%

Statement one alone is sufficient to answer the question.

Statement Two Alone:

The selling price of the television set is $250.

With the information in statement two we can create the following equation:

c + m = 250

With the equation c + m = 250, we can simplify m/(c + m) x 100 as m/250 x 100. However, since we don't know the value of m, we can't determine the value of m/(c + m) x 100. Thus, statement two alone is not sufficient to answer the question.

Answer: A

Scott Woodbury-Stewart
Founder and CEO
[email protected]

Image

See why Target Test Prep is rated 5 out of 5 stars on BEAT the GMAT. Read our reviews

ImageImage